PT 18.S2.Q2

MambaMentality93MambaMentality93 Live Member
edited July 2020 in Logical Reasoning 503 karma

Hey fellow 7Sagers,

I'm having some trouble understanding this question.
Here is how I diagrammed it:

CR ----->/LW
SRO---->LW

(and then I diagrammed the last premise)
L <--S-->CR


SRO -----> /L

I hooked them up by taking the contrapositive of the first premise: SRO ------> LW -------> /CR <--S-->/L
But ultimately, I wasn't able to find the right answer. And JY says that we're supposed to disregard the last premise—Some lawyers are not cattle ranchers L<--S-->CR. How am I supposed to know that I should completely ignore this premise and why? This was the sentence that kept me from getting this question right! I need someone to clarify this concept, please!

Cheers!

Admin Note: https://7sage.com/lsat_explanations/lsat-18-section-2-question-02/

Comments

  • bananabobananabo Core Member
    1211 karma

    I wrote it out this way (which is the same thing that you wrote, but just the contrapositive)

    L ←s→ CR → /LW → /SRO


    L → /SRO

    The “some” statement is seen as irrelevant because it doesn’t do anything to guarantee the conclusion to follow.

    If you have
    X ←s→ Y → Z

    Can you conclude X → Z? No, because there’s no way a “some” statement can conclude an “all” statement. This is why answer choices A and B are wrong, because if you add a “some” statement to this, it doesn’t do anything to allow the conclusion to follow.

    Answer Choice D just says /LW → CR, which also doesn’t allow the conclusion to follow because it’s not linking L to /SRO in any way.

    And answer choice E is irrelevant because it says, “increasing profits → SRO”. This doesn’t link L to /SRO either.

    When I did this, I didn’t disregard L ←s→ CR. I still wrote it out and recognized that in order for the conclusion to follow, I needed the relationship between L and CR to be stronger than a “some” statement and that’s exactly what answer choice C did.

    Hope this helps!

Sign In or Register to comment.